A left Artinian ring that is also a right Noetherian ring

  • Thread starter Thread starter frankusho
  • Start date Start date
  • Tags Tags
    Ring
Click For Summary
A ring that is left Artinian and right Noetherian is proven to be right Artinian. A left Artinian ring has descending chains of left ideals that stabilize, while a right Noetherian ring has ascending chains of right ideals that stabilize. The discussion emphasizes the need for clarity in definitions and the importance of including relevant information when seeking help. Understanding these properties is crucial for concluding the right Artinian status of the ring. The conversation highlights the interconnectedness of these ring properties in abstract algebra.
frankusho
Messages
4
Reaction score
0
1. Prove that a ring which is left Artinian and right Noetherian is right Artinian.



The Attempt at a Solution



I can't figure it out. Can anyone help?
 
Physics news on Phys.org
What are the definitions of these terms?
 
A left Artinian ring R is a ring for which every descending chain R=I0 ⊃I1 ⊃I2 ⊃…⊃In ⊃… of its left ideals stabilizes, i.e. there is a k such that In+1 =In for all n≥k

A right Noetherian ring R is ring in which every ascending chain of right ideals stabalizes
 
So if a ring is left Artinian and right Noetherian, what can you say? What would you like to have happen to conclude that this ring is also right Artinian?
 
When you post a question here, include information like what you have below in your initial post. That's why the 2nd section on relevant equations and definitions is there in the template. It should just be erased.
frankusho said:
A left Artinian ring R is a ring for which every descending chain R=I0 ⊃I1 ⊃I2 ⊃…⊃In ⊃… of its left ideals stabilizes, i.e. there is a k such that In+1 =In for all n≥k

A right Noetherian ring R is ring in which every ascending chain of right ideals stabalizes
 
Question: A clock's minute hand has length 4 and its hour hand has length 3. What is the distance between the tips at the moment when it is increasing most rapidly?(Putnam Exam Question) Answer: Making assumption that both the hands moves at constant angular velocities, the answer is ## \sqrt{7} .## But don't you think this assumption is somewhat doubtful and wrong?

Similar threads

  • · Replies 3 ·
Replies
3
Views
2K
  • · Replies 8 ·
Replies
8
Views
3K
  • · Replies 6 ·
Replies
6
Views
3K
  • · Replies 2 ·
Replies
2
Views
3K
  • · Replies 2 ·
Replies
2
Views
2K
  • · Replies 6 ·
Replies
6
Views
2K
  • · Replies 17 ·
Replies
17
Views
2K
Replies
1
Views
2K
Replies
9
Views
2K
  • · Replies 15 ·
Replies
15
Views
4K